Answered step by step
Verified Expert Solution
Link Copied!

Question

00
1 Approved Answer

d) Suppose r1 = 0.4, r2 = 0.03, d =1,j =150,w = 300 .h = 1000 ,and k = 3000 . Find the equilibrium points

image text in transcribed
d) Suppose r1 = 0.4, r2 = 0.03, d =1,j =150,w = 300 .h = 1000 ,and k = 3000 . Find the equilibrium points of this system. You may do this with just algebra. or use a graphical method (nullclines). or use Sage or a graphing calculator. Note: There is one \"interesting" equilibrium point, that is not on either axis, i.e. for which N and P are both nonzero. e) With the parameters as in part (c) the trajectories approach a limit cycle attractor. Based on this, what can you say about the equilibrium point at which both N and P are nonzero? f) Now, using the same parameters as in part (c) but with r] = 0 ,2, nd the equilibrium points of the system again. By plotting a trajectory or some time series in Sage. what can you say this time about the equilibrium point at which both N and P are nonzero? What phenomenon has occurred between I]: 0.2 and r1: 0 ,4

Step by Step Solution

There are 3 Steps involved in it

Step: 1

blur-text-image

Get Instant Access with AI-Powered Solutions

See step-by-step solutions with expert insights and AI powered tools for academic success

Step: 2

blur-text-image

Step: 3

blur-text-image

Ace Your Homework with AI

Get the answers you need in no time with our AI-driven, step-by-step assistance

Get Started

Recommended Textbook for

Principles of Auditing and Other Assurance Services

Authors: Ray Whittington, Kurt Pany

19th edition

978-0077804770, 78025613, 77804775, 978-0078025617

Students also viewed these Physics questions

Question

Do the measurement items or tools have content relevance?

Answered: 1 week ago